LSAT and Law School Admissions Forum

Get expert LSAT preparation and law school admissions advice from PowerScore Test Preparation.

User avatar
 Dave Killoran
PowerScore Staff
  • PowerScore Staff
  • Posts: 5852
  • Joined: Mar 25, 2011
|
#26963
Complete Question Explanation

The correct answer choice is (A)

In a fortuitous circumstance, this question poses a condition that is met by the hypothetical created in question #3. Thus, we can use that hypothetical to eliminate certain answers such as (B) and (D). However, That said, let us examine this problem as if we did not realize that the hypothetical from question #3 applied to question #4.

If T and X are assigned to the same row (and next to each other), then the other two variables from the S, T, U, X group must be together. Thus, S and U must be in the same row as each other, and we can conclude that S and U must be in the last row (remember, neither can be beside N) and that therefore T and X must be in the front row. Further, from the question stem, T must be seated next to X and another traveler, and so T must be in seat 2. This information is sufficient to prove answer choice (A) correct.

Answer choice (B) is incorrect because if T is assigned to the last row with E, then S and U would be assigned to the first row with N, creating a violation of the last rule.

Answer choice (C) is incorrect because although this could occur in the last row, it does not have to occur. As this is a Must Be True question, this answer is therefore incorrect.

Answer choice (D) is incorrect because O and T must be in different rows, and thus they can never be seated beside each other.

Answer choice (E) is incorrect because although this could occur in the last row, it does not have to occur. As this is a Must Be True question, this answer is therefore incorrect.
 edacyu
  • Posts: 11
  • Joined: Apr 17, 2018
|
#45040
I'm having trouble understanding why O and T must be in separate rules. Could you expand further? It's not in my setup or the one posted for this game.
 Malila Robinson
PowerScore Staff
  • PowerScore Staff
  • Posts: 296
  • Joined: Feb 01, 2018
|
#45050
Hi Edacyu,
To figure out why something cannot happen just try working backwards. So if you are saying you don't understand why O and T can't be in the same row, put them in the same row and see what happens.

According to Rule #1 O has to be in the last row. And Question # 4 puts T next to an unassigned seat and next to a traveler. If you did that and put O & T together in the same row it would mean that the last row was O-T-Empty or Empty-T-O.

The middle row has to be R-P-Empty or Empty-P-R because of the second and third rules.

This leaves S, U and N for the first row. But the fourth rule says that neither S nor U can be next to N. It is impossible to get both S & U away from N if all 3 of them are in the first row.

This means that O cannot be in the same row as T because it made us violate the fourth rule.
Hope that helps!
-Malila
 edacyu
  • Posts: 11
  • Joined: Apr 17, 2018
|
#45142
It does! Thank you so much.
 carolmorales
  • Posts: 3
  • Joined: Oct 30, 2020
|
#81391
Hello,

Could you expand further on why E is incorrect? I chose it because I reasoned that if they are seated next to each other, then that would mean they cannot be in the F row since one of them would inevitably be next to N and that would violate the last rule. What am I missing here? Thanks!
User avatar
 KelseyWoods
PowerScore Staff
  • PowerScore Staff
  • Posts: 1079
  • Joined: Jun 26, 2013
|
#81414
Hi Carol!

If T and X are next to one another, S and U must be in the last row, but they do not have to be next to each other. O could be in between them (S/U O U/S). Thus, answer choice (E) stating that S is assigned to a seat immediately beside U is not something that must be true.

Hope this helps!

Best,
Kelsey

Get the most out of your LSAT Prep Plus subscription.

Analyze and track your performance with our Testing and Analytics Package.